Một số bài toán hình học hay trên báo Toán học tuổi trẻ năm 2016

7 15 0
Một số bài toán hình học hay trên báo Toán học tuổi trẻ năm 2016

Đang tải... (xem toàn văn)

Thông tin tài liệu

Lời giải(Nguyễn Duy Khương) :Trước tiên xin nhắc lại bổ đề quen thuộc không chứng minh: "Cho tam giác ABC nội tiếp đường tròn (O) có X, Y lần lượt thuộc... BC sao cho AX, AY đẳng giá[r]

(1)

Một số tốn hình học hay báo Tốn học tuổi trẻ năm 2016

Tóm tắt nội dung: Báo Toán học tuổi trẻ (THTT) ln nơi hội tụ tốn hay, lời giải đẹp Những năm gần hình học mục "Đề kì này" ln tốn có chất lượng Ở tơi xin giới thiệu số toán số lời giải chúng tới bạn đọc Bài viết đời cận ngày 20-11, xin gửi lời chúc sức khoẻ tới tất thầy cô Đặc biệt thầy tận tình giảng dạy mái trường cấp Amsterdam thân yêu Chân thành biết ơn anh Nguyễn Văn Linh tạo cho nguồn cảm hứng với mơn Tốn chun-đặc biệt mơn Hình học phẳng với giảng thú vị anh

(2)

Lời giải: Trước tiên ta thấy rằng:∠EP F = ∠AP B+∠AP C

2 = 180 ◦−

∠BAC suy ra∠EP F +∠A= 180◦ tứ giác AEP F nội tiếp

Bây ta chứng minh phần lại: "Cho tứ giácAEP F nội tiếp (gócA nhọn P thuộc nửa mặt phẳng bờ EF không chứa A) Phân giác ∠EAP cắt P E I phân giác góc F AP cắt P F L Khi IK qua tâm nội tiếp K tam giác P EF"

Dễ thấy AI qua trung điểm M cung EP AL qua trung điểm N cung F P(xét cung đường tròn AEP F) Mà K tâm nội tiếp tam giác P EF M, K, F thẳng hàng N, K, E thẳng hàng Đến áp dụng định líP ascalcho lục giác nội tiếpAEM P N F suy ngayI, K, L thẳng hàng

Bài toán 2(Hồ Quang Vinh-THTT-T8/466-tháng 4/2016): Cho tam giác ABC có đường cao AH Đường trịn tâm I đường kính AH cắt AB, AC S, T BT cắt CS K Gọi tiếp tuyến S T (I) cắt M M A giao BC N IM cắt ST J Chứng minh N J qua trung điểm AK

(3)

Trước vào lời giải ta cần chứng minh bổ đề nhỏ sau: "Cho tam giác ABC ngoại tiếp đường tròn(I) (I)tiếp xúcBC, CA, AB tạiD, E, F DM đường kính của(I) Tiếp tuyến (I)tại M cắtDF, DE Gvà H Gọi ADcắt GH K Khi đóK trung điểm GH."

Chứng minh: Gọi GH cắt AB, AC J, N Gọi AM cắt BC P Ta dễ thấy rằng: P C = BD Theo định lí Tháles ta lại có rằng: GJ

DB = F J F B =

F J BD ⇒ GJ = J F = J M Hồn tồn tương tự ta có: N M = N H = N E Lại theo định lí Tháles ta có: J K

BD = M N

P C (= AK

AD) mà ta có BD = P C nên ta thu rằng: J K = M N Do GJ +J K = J M +M N = J N KH =KM +M N +N H =J K+M N+KM =J N đóGK =KH(= J N) hay K trung điểm GH

(4)

Bài toán 3(Nguyễn Văn Linh-T12/468-THTT-tháng 6/2016): Cho tứ giác ngoại tiếp ABCD có đường tròn nội tiếp (I) Tia AB cắt tia CD điểmE tia DA cắt tia CB điểm F Gọi I1, I2 tâm nội tiếp tam giác EF B tam

giác EF D Chứng minh rằng: ∠I1IB=∠I2ID

Lời giải(Nguyễn Duy Khương): Ta ý tia AB cắt tia CD điểm E thuộc nửa mặt phẳng bờ AD chứa C ta có: DI DI2 phân giác ∠EDF

đó E, I, I2 thẳng hàng Tương tự F, I, I1 thẳng hàng Do hiển nhiên

góc cần chứng minh suy biến thành góc bẹt

Vậy nên ta cần chứng minh trường hợp tia AB cắt tiaCD E nằm bờ nửa mặt phẳngBC chứaD Xin nêu bổ đề phụ (Định lí Pythot suy rộng cho tứ giác lõm): "Cho tứ giác ABCD ngoại tiếp đường tròn (I) Giả sử tia AB cắt CD E tia DA cắt tiaBC điểm F Khi ta có: ED−F B =EB−F D" (Bạn đọc tự chứng minh)

Quay trở lại toán, gọi K, L hình chiếu I2và I1 lên ED F B

Gọi X, Y, Z, T tiếp điểm (I) với AB, BC, CD, DA Do I2D

phân giác ngồi góc F DC nên I2D ⊥ ID tương tự ∠I1BI = 90◦ Do đpcm

tương đương với: 4I2ID ∼ 4I1IB hay

I2D

DI = I1B

BI Ta có: ∠KI2D = 90 ◦ − ∠F DE

2 =

∠F DC

(5)

tự ta có:4I1BL ∼ 4BIY(g.g) nên ta thu từ rằng:

I2D

ID = DK

IZ I1B

IB = BL

IY đpcm tương đương với: DK = BL(nhờ IY = IZ) Ta dễ thấy KD = DF +DE−EF

2 LB =

EB+F B −EF

2 đpcm tương đương với DE−F B =EB−F D(đúng theo bổ đề) Vậy ta thu đpcm

Nhận xét: Bài toán toán mà ấn tượng việc sử dụng tam giác đồng dạng đối xứng để cuối đưa hệ thức P ythot tinh tế

Bài toán 4(Lê Viết Ân-T12/470-THTT-tháng 8/2016): Cho tam giác ABC có H nằm tam giác thoả mãn: ∠HBA = ∠HCA Một đường tròn qua B, C cắt (AH) điểm X, Y Gọi AH∩BC =D, gọi K hình chiếu D lên XY Chứng minh rằng: ∠BKD =∠CKD

Lời giải(Nguyễn Duy Khương): Trong lời giải ta quy ước(XY Z) đường tròn ngoại tiếp tam giác XY Z (U V) đường trịn đường kínhU V

(6)

nội tiếp Gọi I tâm đường tròn (BCF E), gọi (AF E) cắt lại (O) điểm thứ hai L 6= A AL, EF, BC trục đẳng phương cặp đường tròn đơi giao nhau: (AEF),(O); (I),(AEF); (I),(O) AL, EF, BC đồng quy điểm (gọi S).Gọi P, Q hình chiếu H lên AB, AC Dĩ nhiên ta có: 4HP B ∼ 4HQC(g.g) Vậy: P B

QC = HB HC =

EB

F C (do 4HEB ∼ 4HF C) lại có rằng: (LB, AB) ≡ (LC, AC)(modπ),(LE, BE) ≡ (AE, LE) ≡ (LF, F C)(modπ) nên 4LEB ∼ 4LF C(g.g) Vậy LB

LC = F B F C =

P B

QC hiển nhiên 4LBP ∼

4LCQ(c.g.c) suy (LP, BP) ≡ (LQ, QC) hay (LP, AP) ≡ (LQ, AQ)(modπ) hay L ∈ (AH) Vậy AL trục đẳng phương (AH) (AEF), XY trục đẳng phương của(AH)và(I),EF trục đẳng phương của(I)và(AEF)nênXY, EF, AL đồng quy XY, EF, AL, BC đồng quy S Theo hàng điểm điều hoà (SD, BC) = −1 theo phép chiếu xuyên tâm K K(SD, BC) = K(Y D, BC) = −1 màKD ⊥XY KDlà phân giác∠BKD ta có được: ∠BKD=∠CKD(đpcm)

Bài tốn 5(Nguyễn Xuân Hùng-THTT số 471-tháng 9/2016)):Cho tam giác ABC có I tâm nội tiếp Một đường thẳng d qua I vng góc AI Lấy điểm E,F thuộc d cho ∠EBA=∠F CA= 90◦ Các điểm M, N nằm BC cho M E kN F kAI Chứng minh rằng: (ABC)tiếp xúc (AM N)

(7)

BC cho AX, AY đẳng giác Khi tam giác (AXY)tiếp xúc (ABC)." Quay trở lại toán:

Trường hợp 1(AB = AC) Khi ta có: 4IAB = 4IAC đồng thời thấy rằng: EF M N hình chữ nhật từ hiển nhiên ta có: AM, AN đẳng giác tam giác ABC biến đổi góc đơn giản Do theo bổ đề ta có đpcm

Trường hợp 2(AB 6=AC) Gọi P, Qlần lượt trung điểm cung AB AC không chứa C, B (O) Từ giả thiết ta có rằng: A, B, E, I đồng viên nên ta có: P tâm(AEBI)(do ta có kết quen thuộcP tâm(AIB))(∗) Hồn tốn tương tự thìQ tâm(AICF) (∗∗) Gọi BI,CI cắt lại(AIC) (AIB) điểmK, LkhácI Từ (∗)(∗∗)ta thu hình chữ nhật AIEL AIF K Lại có: M E k N F k AI nên thu được: L, E, M thẳng hàng K, F, N thẳng hàng Ta ý AB6=AC nên LK không song song BC

Hiển nhiên từ hình chữ nhật ta thấy rằng: ∠LAI = ∠KAI = 90◦ LAK ⊥ AI Gọi AD đường phân giác góc BAC, gọi LK cắt BC điểm J mà LAK ⊥AI nên AJ phân giác ngồi góc BAC

Ta thấy rằng: AD, BK, CL đồng quy I Do áp dụng tính chất hàng điểm điều hồ (J ALK) = (J DBC) = −1 Kẻ AH vng góc BC điểm H Ta thấy rằng: H(J ALK) = −1 mà HA ⊥ HJ HA phân giác góc LHK Vậy mà lại có: A, L, M, K đồng viên vàA, H, N, K đồng viên(lần lượt thuộc đường trịn đường kính AM AN) nên ta có: (M L, M A) ≡ (HL, HA) ≡ (HK, HA) ≡

(N K, N A)(modπ) ý M E k N F k AI AI phân giác góc M AN hay làAM, AN đẳng giác tam giác ABC Áp dụng bổ đề ta có(AM N) tiếp xúc (ABC)(đpcm)

Ngày đăng: 09/02/2021, 02:44